Consider all pairs of real numbers such that a + b = 3 .
What is the minimum value of
a 2 b 2 − 2 a 2 b − 2 a b 2 + a 2 + 4 a b + b 2 − 2 a − 2 b + 1 ?
This section requires Javascript.
You are seeing this because something didn't load right. We suggest you, (a) try
refreshing the page, (b) enabling javascript if it is disabled on your browser and,
finally, (c)
loading the
non-javascript version of this page
. We're sorry about the hassle.
The expression is equal to ( a − 1 ) 2 ( b − 1 ) 2 .
The minimum is clearly 0, and occurs when { a , b } = { 1 , 2 } .
Note: Some may think that the minimum occurs when a = b = 2 3 , which will yield the value 0.0625. Why doesn't this work?
3 a + 3 b = 1
A triangle is formed ,
A r e a = f ( a ) = 2 1 a ( 3 − a )
f ′ ( a ) = 3 − 2 a
f ′ ( a ) = 0 , a = 2 3
checking whether it is the point of maxima of minima ,
f ′ ′ ( a ) = − 2 thus for a = 2 3 , the area obtained is maximum. That's why it does'nt work here.
Sir how would we find the point of minima using derivatives?
I took the expression to (ab-2)^{2} and the a=1 or b=2 which gave the minimum at 0.
Log in to reply
Right, because ( a − 1 ) ( b − 1 ) = a b − a − b + 1 = a b − 2 .
Suppose n = a + b = 3 and m = a b . Then the given is m 2 − 2 m n + n 2 + 2 m − 2 n + 1 . Subbing n = 3 we end up with m 2 − 4 m + 4 = ( m − 2 ) 2 . Since m ≤ n 2 / 4 = 2 . 2 5 by AM-GM, we can take m = 2 which yields the minimum, 0 .
Equality occurs at the roots of x 2 − 3 x + 2 by Vieta's, which are 1 , 2 .
Qwswhejjnjhhbwjjkkskk
a 2 b 2 − 2 a b ( a + b ) + ( a + b ) 2 − 2 a b + 4 a b − 2 ( a + b ) + 1 ( a b ) 2 − 2 a b ( 3 ) + 9 + 2 a b − 6 + 1 the minimum value of ab >0 is a+b-1=2{note that zero wont work} 2 2 − 4 × 3 + 9 + 4 + 1 − 6 4 − 1 2 + 1 4 − 6 0
Problem Loading...
Note Loading...
Set Loading...
The expression can be written as
a 2 b 2 − 2 a 2 b − 2 a b 2 + 2 a b + ( a + b − 1 ) 2
Since a + b = 3
a 2 b 2 − 2 a b ( a + b − 1 ) + 4
a 2 b 2 − 4 a b + 4
Now let a b = t
Differentiating the function
t 2 − 4 t + 4
we get
2t - 4 = 0
t = 2
we have to check if ab = 2 valid ( given a + b = 3)
For (a,b) = {1,2} it satisfies the given condition hence
a b m i n = 2
a 2 b 2 − 4 a b + 4 = 0